Đến nội dung

lethanhtuan213

lethanhtuan213

Đăng ký: 07-01-2017
Offline Đăng nhập: 12-07-2019 - 14:18
*****

#721218 Có tồn tại hay không hàm số $f:N^{*}\rightarrow N^{*...

Gửi bởi lethanhtuan213 trong 01-04-2019 - 21:38

Giả sử tồn tại hàm f thỏa mãn bài toán thì ta có:
f : đơn ánh.
Với
f(n1) = f(n2) => f(m.f(n1)) = f(m.f(n2)) => n1 + f(2018m) = n2 + f(2018m) => n1 = n2. => f : đơn ánh.
Thay m = 1 => f(f(n)) = n + a ( a = f(2018)
m=1 => f toàn ánh
Thay m = 1, n = f(n) => f(n+a) = f(n) + a => f(n+ma) = f(n) + a.m
Thay n = ma => f(mf(ma)) = ma + f(2018m) = f(ma+2018m) => mf(ma) = ma + 2018m => f(ma) = a + 2018 .
Thay n = 2018 => f(ma) = 2018 + f(2018m) => f(2018m) = a => f(2018m1) = a = f(2018m2) với m1 khác m2 => vô lí.
vậy không tồn tại hàm f thỏa mãn đề bài




#719420 ĐỀ THI HSG TOÁN 9 THÀNH PHỐ HÀ NỘI

Gửi bởi lethanhtuan213 trong 13-01-2019 - 11:13

T52VKKW.jpg




#711400 Đề thi vào lớp 10 THPT Chuyên Quảng Nam năm 2018

Gửi bởi lethanhtuan213 trong 22-06-2018 - 15:19

Câu 6 Dùng khai triển S.O.S
P/S: Có cách khác không các bạn

BDT.jpg

Cách của mình




#709996 ĐỀ THI VÀO LỚP 10 CHUYÊN TOÁN LÊ QUÝ ĐÔN - ĐÀ NẴNG 2018-2019

Gửi bởi lethanhtuan213 trong 05-06-2018 - 12:38

yfuoQ7X.jpg




#709379 Đề thi vào 10 chuyên Lê Hồng Phong - Nam Định 2018 - 2019 - Toán Chuyên

Gửi bởi lethanhtuan213 trong 27-05-2018 - 18:34

Câu 3c. $O_{1}H$ và $O_{2}H$ là 2 tia phân giác của hai góc kề bù => $O_{1}H$ vuông góc với $O_{2}H$ . 
Có $\Delta HO_{1}M đồng dạng với \Delta CO_{2}H(g.g)$ => $\frac{O_{1}H}{O_{2}H}=\frac{MH}{CH}=\frac{MB}{MC}=>\Delta O_{1}HO_{2} đồng dạng với \Delta BMC$ => $\frac{S_{O_{1}HO_{2}}}{S_{BMC}}=(\frac{O_{2}H}{CM})^{2}$ Mà $\frac{O_{2}H}{CM}=\sqrt{2}\frac{CH+MH-CM}{CM}\leq \sqrt{2}(\frac{\sqrt{2(MH^{2}+CH^{2})}-CM}{CM}=\sqrt{2}(\sqrt{2}-1)$ => $S_{O_{1}HO_{2}}\leq P_{BMC}\sqrt{2}(\sqrt{2}-1)$ Mà $P_{BMC} = BM+CM+BC  $\leq$  căn 2(BM^2+CM^2)+BC = \sqrt{2} x 2R+2R  => $P_{O_{1}HO_{2}}\leq 2\sqrt{2}(\sqrt{2}+1)(\sqrt{2}-1)R=2\sqrt{2}R  . Dấu "=" <=> M chính giữa




#709376 Đề thi vào 10 chuyên Lê Hồng Phong - Nam Định 2018 - 2019 - Toán Chuyên

Gửi bởi lethanhtuan213 trong 27-05-2018 - 18:20

câu 3.

a. Chứng minh được $BMKE$ nội tiếp.

suy ra $\angle BEC = \angle BKC = \angle BAE \rightarrow BE^2 = BC.BA$.

b. theo hệ thức lượng $BE^2 = BC.BA = BN^2$ kết hợp với $\angle BNP = \angle BAP = \angle BEP \rightarrow dpcm$.

c. $P_{O_1HO_2} = O_1O_2 + O_1H +  O_2H = \frac{1}{2}P_{CAB} \rightarrow P_{O_1HO_2}$ max khi $P_{CAB}$ max tức là $C$ là điểm chính giữa nửa đường tròn đường kính $BC$.

attachicon.gifdiendan(132).PNG.

Câu c là tâm đường tròn nội tiếp chứ không phải ngoại tiếp bạn




#707893 Chứng minh bất đẳng thức: Với a,b,c thuộc khoảng 0;2

Gửi bởi lethanhtuan213 trong 08-05-2018 - 10:20

wB7C6sX.jpg




#706298 ĐỀ THI HSG TOÁN 9 TỈNH QUẢNG NAM 2017-2018

Gửi bởi lethanhtuan213 trong 18-04-2018 - 16:18

3)b) Ta có:$b^{2}-a^{2}=p^{2}<=>(b-a)(a+b)=p^{2}$

$=>b-a,a+b$ là ước của $p^{2}$

Do $a+b> b-a> 0=>\left\{\begin{matrix}b-a=1 \\ a+b=p^{2} \end{matrix}\right. <=> \left\{\begin{matrix}b=a+1 \\ 2a+1=p^{2} \end{matrix}\right.$

$=>2(p+a+1)=p^{2}+2p+1=(p+1)^{2}$ là SCP.

Lại có, $\left\{\begin{matrix}b=a+1 \\ p^{2}=a+b=2a+1 \end{matrix}\right.$

$=>2a=p^{2}-1$

Do $p$ là số nguyên tố lớn hơn 3 $=>p^{2}-1\vdots 3$

+)Nếu $p=4k+1=>a=8k(k+1)\vdots 4$

+)Nếu $p=4k+3=>a=8(k+1)(k+2)\vdots 4$

$=>a\vdots 12$

Bạn làm giống mình này :D 




#706297 ĐỀ THI HSG TOÁN 9 TỈNH QUẢNG NAM 2017-2018

Gửi bởi lethanhtuan213 trong 18-04-2018 - 16:15

Đề năm nay hơi dễ so với mọi năm 




#706281 ĐỀ THI HSG TOÁN 9 TỈNH QUẢNG NAM 2017-2018

Gửi bởi lethanhtuan213 trong 18-04-2018 - 12:33

fgj5o2U.jpg




#705579 BẤT ĐẲNG THỨC

Gửi bởi lethanhtuan213 trong 12-04-2018 - 11:13

$\frac{ab^{3}}{a^{3}+b^{3}}+\frac{bc^{3}}{b^{3}+c^{3}}+\frac{ca^{3}}{c^{3}+a^{3}}\leq \frac{a+b+c}{2}$

 

NHỜ MỌI NGƯỜI GIÚP MÌNH BÀI NÀY VỚI Ạ!




#686059 $\boxed{\text{TOPIC}}$ Ôn thi học si...

Gửi bởi lethanhtuan213 trong 30-06-2017 - 21:00

Tiếp tục nhé :

4 CMR Nếu $m,n$ là các STN thỏa mãn$4m^{2}+m=5n^{2}+n$

thì $m-n$ và $5m+5n+1$ đều là số chính phương

5.Cho x,y,z là các STN thỏa mãn

$(x-y)(y-z)(z-x)=x+y+z$

CMR $xyz\vdots 27$

6. GPT $x^{2}y^{2}-x^{2}-8y^{2}=2xy(x,y \in Z)$

 

Câu 4: $4m^{2}+m=5n^{2}+n \Leftrightarrow 5m^{2}-5n^{2}+m-n = m^{2}\Leftrightarrow (m-n)(5m+5n+1)=m^{2}$ (*)
Giờ thì cần c.m (m-n,5m+5n+1)=1 nữa là xong
Gọi (m-n,5m+5n+1)=d
Ta có: $m - n \vdots d , 5m+5n+1 \vdots d. (*)=> n^{2}\vdots d^{2} =>n\vdots d$
$=> m \vdots d => n\vdots d => 5(m+n) \vdots d => 1 \vdots d$ => ĐpCM 




#686045 $\boxed{\text{TOPIC}}$ Ôn thi học si...

Gửi bởi lethanhtuan213 trong 30-06-2017 - 20:09

Bạn ơi mình nghĩ nên sửa tiêu đề lại là: 2017-2018 chứ :) 




#686041 $\boxed{\text{TOPIC}}$ Ôn thi học si...

Gửi bởi lethanhtuan213 trong 30-06-2017 - 19:57

Câu 3: $a^{2}+5b^{2}-(3a+b)\geq 3ab-5 \Leftrightarrow a^{2}+5b^{2}-(3a+b)-3ab+5\geq 0 \Leftrightarrow 2a^{2}+10b^{2}-6ab-6a-2b+10\geq 0 \Leftrightarrow (a-3b)^{2}+(a-3)^{2}+(b-1)^{2} \geq 0$
Bất đẳng thức cuối hiển nhiên đúng => ĐPCM. Dấu bằng xảy ra <=> a=3 , b = 1




#686040 $\boxed{\text{TOPIC}}$ Ôn thi học si...

Gửi bởi lethanhtuan213 trong 30-06-2017 - 19:54

Câu 2:
$n^{3}+n^{2}+1 = n.n(n+1)+2 \vdots 2$
mà n là số tự nhiên lớn hơn 0 => ĐPCM